OG 10 problem

This topic has expert replies
User avatar
Master | Next Rank: 500 Posts
Posts: 307
Joined: Wed Feb 22, 2012 9:45 pm
Thanked: 12 times
GMAT Score:700

OG 10 problem

by Gaurav 2013-fall » Fri May 18, 2012 11:52 pm
When people evade income taxes by not declaring taxable income, a vicious cycle results. Tax evasion forces lawmakers to raise income tax rates, which causes the tax burden on nonevading taxpayers to become heavier. This, in turn, encourages even more taxpayers to evade income taxes by hiding taxable income.The vicious cycle described above could not result unless which of the following were true?

(A) An increase in tax rates tends to function as an incentive for taxpayers to try to increase their pretax
incomes.
(B) Some methods for detecting tax evaders, and thus recovering some tax revenue lost through evasion, bring
in more than they cost, but their success rate varies from years to year.
(C) When lawmakers establish income tax rates in order to generate a certain level of revenue, they do not allow adequately for revenue that will be lost through evasion.
(D) No one who routinely hides some taxable income can be induced by a lowering of tax rates to stop hiding
such income unless fines for evaders are raised at the same time.
(E) Taxpayers do not differ from each other with respect to the rate of taxation that will cause them to evade
taxes.


OA C

Legendary Member
Posts: 2789
Joined: Tue Jul 26, 2011 12:19 am
Location: Chennai, India
Thanked: 206 times
Followed by:43 members
GMAT Score:640

by GmatKiss » Sat May 19, 2012 1:16 am
A/C; C is better

Master | Next Rank: 500 Posts
Posts: 341
Joined: Sun Mar 25, 2012 6:59 pm
Thanked: 17 times
Followed by:4 members
GMAT Score:720

by ice_rush » Sat May 19, 2012 12:39 pm
if choice (A) were true then the vicious cycle would probably not occur in the first place..so that is definitely not the assumption.

on the contrary (C) is the necessary assumption for the argument to be true. negate (C) and the argument falls apart.


hope this helps
Last edited by ice_rush on Sun May 20, 2012 9:34 am, edited 1 time in total.

User avatar
GMAT Instructor
Posts: 15539
Joined: Tue May 25, 2010 12:04 pm
Location: New York, NY
Thanked: 13060 times
Followed by:1906 members
GMAT Score:790

by GMATGuruNY » Sun May 20, 2012 4:05 am
Gaurav 2013-fall wrote:When people evade income taxes by not declaring taxable income, a vicious cycle results. Tax evasion forces lawmakers to raise income tax rates, which causes the tax burden on nonevading taxpayers to become heavier. This, in turn, encourages even more taxpayers to evade income taxes by hiding taxable income.The vicious cycle described above could not result unless which of the following were true?

(A) An increase in tax rates tends to function as an incentive for taxpayers to try to increase their pretax
incomes.
(B) Some methods for detecting tax evaders, and thus recovering some tax revenue lost through evasion, bring
in more than they cost, but their success rate varies from years to year.
(C) When lawmakers establish income tax rates in order to generate a certain level of revenue, they do not allow adequately for revenue that will be lost through evasion.
(D) No one who routinely hides some taxable income can be induced by a lowering of tax rates to stop hiding
such income unless fines for evaders are raised at the same time.
(E) Taxpayers do not differ from each other with respect to the rate of taxation that will cause them to evade
taxes.

OA C
The premise is about X: People EVADE income taxes.
The conclusion is about Y: Lawmakers will be forced to RAISE INCOME TAX RATES.

The argument assumes that X causes Y: that TAX EVASION causes lawmakers to RAISE TAXES.

Answer choice C, negated: When lawmakers establish income tax rates, they allow adequately for revenue that will be lost through evasion.

If lawmakers establish tax rates that allow for tax evasion, then the conclusion that tax evasion will force lawmakers to raise taxes is invalidated.
Since the negation of C invalidates the conclusion, C is the necessary assumption: WHAT MUST BE TRUE for the conclusion to be valid.

The correct answer is C.
Private tutor exclusively for the GMAT and GRE, with over 20 years of experience.
Followed here and elsewhere by over 1900 test-takers.
I have worked with students based in the US, Australia, Taiwan, China, Tajikistan, Kuwait, Saudi Arabia -- a long list of countries.
My students have been admitted to HBS, CBS, Tuck, Yale, Stern, Fuqua -- a long list of top programs.

As a tutor, I don't simply teach you how I would approach problems.
I unlock the best way for YOU to solve problems.

For more information, please email me (Mitch Hunt) at [email protected].
Student Review #1
Student Review #2
Student Review #3

User avatar
Master | Next Rank: 500 Posts
Posts: 148
Joined: Sun Apr 29, 2012 1:08 pm
Thanked: 8 times

by jimmyjimmy » Sun May 20, 2012 7:19 am
why the answer choice is made dark, i cannot see, why this is so??

User avatar
Junior | Next Rank: 30 Posts
Posts: 24
Joined: Sat Feb 06, 2010 6:13 pm

by ramprakaashk » Wed Jun 06, 2012 8:46 pm
Move your cursor over the dark answer choices and you will see the answers

User avatar
Junior | Next Rank: 30 Posts
Posts: 24
Joined: Sat Feb 06, 2010 6:13 pm

by ramprakaashk » Wed Jun 06, 2012 8:49 pm
Can some one describe the answer choice : D and tell why D is wrong.

User avatar
Master | Next Rank: 500 Posts
Posts: 148
Joined: Sun Apr 29, 2012 1:08 pm
Thanked: 8 times

by jimmyjimmy » Mon Jun 11, 2012 8:45 am
one tip for CR's.
avoid answer choices that are too firm and solid (no one for the D choice)

D says ( no one will hide the tax return if tax % is reduced )
there is a possibility that ssm1 will hide..even after reducing the rates (i wd b dh fst one )

and too firm (no one)

hopt it helps!!